Difference between revisions of "2021 AMC 12B Problems/Problem 1"

(Solution)
(Solution 2)
Line 9: Line 9:
 
<br><br>
 
<br><br>
 
~ {TSun} ~
 
~ {TSun} ~
 +
 +
== Video Solution by OmegaLearn (System of Equations) ==
 +
https://youtu.be/hyYg62tT0sY

Revision as of 20:48, 11 February 2021

How many integer values of $x$ satisfy $|x|<3\pi$?

$\textbf{(A)} ~9 \qquad\textbf{(B)} ~10 \qquad\textbf{(C)} ~18 \qquad\textbf{(D)} ~19 \qquad\textbf{(E)} ~20$

Solution

Since $3\pi$ is about $9.42$, we multiply 9 by 2 and add 1 to get $\boxed{\textbf{(D)}\ ~19}$~smarty101

Solution 2

$|x|<3\pi$ $\iff$ $-3\pi<x<3\pi$. Since $\pi$ is approximately $3.14$, $3\pi$ is approximately $9.42$. We are trying to solve for $-9.42<x<9.42$, where $x\in\mathbb{Z}$. Hence, $-9.42<x<9.42$ $\implies$ $-9\leq x\leq9$, for $x\in\mathbb{Z}$. The number of integer values of $x$ is $9-(-9)+1=19$. Therefore, the answer is $\boxed{\textbf{(D)}19}$.

~ {TSun} ~

Video Solution by OmegaLearn (System of Equations)

https://youtu.be/hyYg62tT0sY